Search This Blog

Tuesday, June 8, 2010

Possible Lab Final Scenarios

Double/Single Slit
Diffraction Grating
LRC/RL CIRCUITS!!!!!!!
OSCILLISCOPE LAB!!!!
Lenses
Emission Spectra

review and memorize the circuit diagram for the LRC/oscilliscope/function generator lab!!

Good luck guys! especially Lauren cause you actually read these

Tuesday, June 1, 2010

just in case anyone cares

I'm sure you chem people know about the Aufbau Principle which basically states that electrons fill in the lower energy levels first and then move their way up from there. It is one of the most basic theories that is the basis for all of chemistry. Well, the Stanford Particle Accelerator Lab has just discovered with their X-ray picoscope (which is an actual thing I didn't make it up it means a "microscope" that uses particles with a wavelength of 10^-12) which can actually watch chemical reactions occur at the atomic level, that they in fact do not follow this principle, but randomly fill in whatever orbital is available...so in conclusion Chem sucks; Physics rocks!

Quantum Numbers cont'd

So in order to use this procedure with particular atoms we have to start with Hydrogen because we know that Hydrogen has 1 electron, which is in the n = 1 shell. So beginning with hydrogen:
Element: Hydrogen, # of electrons = 1
n = 1, l = 0, m_l = 0, m_s = +1/2 (or you could write n = 1, l = 0, m_l = 0, m_s = -1/2), but since there is only one electron you only write one of these two options.
Now as we go down and continue to add these numbers the number of electrons will coincide with each element. The next one being Helium because it has two electrons:
Element: Helium, # of electrons = 2
n = 1, l = 0, m_l = 0, m_s = +1/2
n = 1, l = 0, m_l = 0, m_s = -1/2
Then, you continue down the line:
Element: Li, # of electrons = 3
n = 1, l = 0, m_l = 0, m_s = +1/2
n = 1, l = 0, m_l = 0, m_s = -1/2
n = 2, l = 0, m_l = 0, m_s = +1/2

So basically you just list everything in order and for each electron you count them and put the atom by the # of electrons it has

So write the configurations for Oxygen

Quantum Numbers

Ok so it sounds like some of you are confused about quantum numbers so here we go:
Each set of quantum numbers, which will be a set of four numbers separated by commas, represents one electron and said electrons "position" in the atom. According to the Pauli Exclusion Principle, which you should know, no two electrons can have the same 4 quantum numbers. The quantum numbers each have rules discussing which numbers they are "allowed" to be, which are as follows:
n = 1, 2, 3, 4,...,infinity
l = 0, 1, 2, 3, ... , n - 1
m_l = -l , ... ,+l
m_s = -0.5, +0.5
Therefore if I have a certain n number like n = 3 I can have electrons with angular momentum numbers, l numbers, up to n - 1; which for us is 3 - 1 = 2. Thus, the L-numbers possible are l = 0, 1, 2 and thats it. IN EACH OF THOSE I can have a magnetic quantum number, m_l-number, going from -l to +l. Then in each of those we can have an electron spin of -1/2 or +1/2. So for n = 3 we have these possible electron positions:
n = 3, l = 0, m_l = 0, m_s = +1/2
n = 3, l = 0, m_l = 0, m_s = -1/2
notice that these two electrons have all the same quantum numbers except for the m_s number, or the electron spin number. Continuing on:
n = 3, l = 1,
now since l = 1 my m_l numbers can either be -1, 0, +1
n = 3, l = 1, m_l = -1,
now there are two more possibilities for m_s:
n = 3, l = 1, m_l = -1, m_s = -1/2
n = 3, l = 1, m_l = -1, m_s = +1/2
and when m_l = 0:
n = 3, l = 1, m_l = 0, m_s = -1/2
n = 3, l = 1, m_l = 0, m_s = +1/2
and finally when m_l = +1:
n = 3, l = 1, m_l = +1, m_s = -1/2
n = 3, l = 1, m_l = +1, m_s = +1/2

Now try and write all of the configurations for when n = 3, l = 2


Monday, May 31, 2010

Studying?

I'm starting to worry because no one is writing any questions on here and no one has tried to contact me with questions....are any of you studying?

Saturday, May 29, 2010

Problem 32.3 (Assignment 14)

Question: What is the nuclear radius of 197,79 Au?

Gold (Au)
Atomic Number: 79
Atomic Mass: 197

I figured out the answer to be 7*10^-15 m

but the answer is in meters, and mastering physics wants it in fm.
how do I convert it to fm?

Thursday, May 27, 2010

Final Review Session

The final review session will be on Friday, June 4th from 8am - 12:00pm. If there are any other times you would like to request then please just let me know. Thanks
Dante

Conceptual Questions

1. What happens to the electric field and the capacitance when a dielectric is added to a capacitor? Why? Draw a diagram to illustrate your explanation.

2. Compare and/or contrast electric potential energy and electric potential. Conceptually, is there a difference or are these just two names for the same thing? Explain.

3. On the passenger side all automobiles the side mirror states “object are closer than they appear.” What type of mirror is used on the passenger side? Why is it necessary to have this warning?

4. What is resonance frequency? What are its effects on the elements in an RLC circuit as well as the entire circuit itself?

5. Can anything go faster than the speed of light? Explain.

6. What is the twin paradox.

7. What is the result, as predicted by geometric optics, of light going through a double slit? Is this prediction correct? Explain.

8. What does the “phase” of a circuit mean?

9. Explain, at the subatomic level, what is happening inside the gas tubes that you used for the line spectra lab?

10. a) Compare and/or contrast fission and fusion. Conceptually, is there a difference or are these just two names for the same thing? Explain. b) Which, if any, can be utilized to create energy by humans?

11. Explain how a nuclear bomb works in terms of the radioactive source. Why does this process release SOOO much energy?

12. What is an electron-volt? Why is it used?

13. a) Compare and/or contrast the Bohr model of the atom and the quantum-mechanical model of the atom. Conceptually, is there a difference or are these just two names for the same thing? Explain. b) Where and how are each used?

14. What is an interferometer? What does this device tell us about light?

Exam III and Final Review

Suggested Problems: 19-31, 19-79,20-33, 20-57, 21-45, 21-60*, 21-93, 22-47(764), 22-65 (766), 22-73 (766), 22-79 (767), 22-83 (767), 23-25 (799), 23-30 (799), 23-67(801), 23-81(802), 24-79(836), 24-69(835), 24-59(835), 25-57(867), 25-97 (869), 26-35, 26-55, 26-6126-101, 27-51, 27-69, 27-89, 28-71 (968), 28-63(968), 28-79 (969), 29-77, 29-63, 30-77(1033), 30-81(1033), 31-63(1070), 31-70(1070), 31-31(1068), 32-21 (1107), 32-40(1108), 32-51(1108), 32-77(1109), 32-65(1109), 32-71 (1109)

*a dielectric has been added to the 9.50 mF capacitor with k = 3.2


Thursday, May 20, 2010

CH 30 & 31 Practice Problems

Dante assigned the following practice problems in SI for chapters 30 & 31

CHAPTER 30:
# 55, 73, 78, 81

CHAPTER 31:
# 21, 31, 63

=)

Monday, May 10, 2010

Chapter 28 Practice Problems

28-5, 28-17, 28-21, 28-36, 28-44, 28-63, 28-71, 28-79

Sunday, May 9, 2010

New Practice Problems

26-93, 27-51, 27-69, 27-89, 27-75, 27-43, 27-27

Monday, May 3, 2010

If you were thinking of doing this...

Tuesday

Hey guys, sorry I missed class but if any of you are available I will be having a makeup session tomorrow, Tuesday April 4th at 4:10p. If you want to come just meet me in 60-1501. Has anyone been keeping up on what we talked about last week?

Friday, April 30, 2010

Practice Problems

26.51, 26.89, 26.85, 26.101, 26.95, 26.97, 26.61

Thursday, April 29, 2010

Practice Problems

I hope you all are doing your practice problems. Could someone post which ones I said to do please. If anyone is stuck I highly suggest that you discuss them here.

Sunday, April 25, 2010

For Tonight

I have just finished posting all of the answers to the sample exam. I am going to keep the blog open for the rest of the night so feel free to comment and I will respond pretty quickly

25.8

1.19E7 m/s away from the Earth

24.6

39.2 degrees and assuming the omega is moving CCW ImaxZ should be behind ImaxR

24.5

L = 0.608 mH
R = 6.52 k(Ohms)

24.4

a) Z = 606 Ohms
b) increase (if you get something different THAT'S OK!!! tell me why you reasoned that way)

23.5

a) L = 4.22 H
b) 0.28 s
c) 1.64 A

25.6

49 degrees

25.7

3.85*10^26 W

25.5

a) 8.238*10^14 Hz
b) 8.203*10^14 Hz

24.3

a) 0.173 V
b) 7.84 V
c) 1.84 V

23.7

b) clockwise
c) (mgR)/(L^2)(B^2)

23.6

4.0 mWb

22.4

At A - 7.8 micro T (out of the page)
At B - 0
At C - 7.8 micro T (into the page)

22.3

arctan[(ILB)/(mg)]

22.2

up and to the left

All Conceptual Problems

post your answers and I will correct them

I Lied

Ok I just got home so I am going to post the answers to the sample exam as they become available

sinusoidal & square waves

what is the relationship between sinusoidal and square waves? just curious..

Saturday, April 24, 2010

Sample Exam Answers

Sorry guys I have been out of town since Thursday morning and this is the first chance I have had to get onto the blog. I will be getting back at around 3pm tomorrow so I will try and post the answers ASAP and answer all of your questions that you posted around that time.

As#9: Polarization of Light and Malus's Law (PART F)

The question reads:

Notice that a polarizer modifies the light intensity according to Malus's law and also reorients the polarization angle of the beam to match its own transmission axis. Hence it is possible for light to pass through a pair of crossed polarizers if a third polarizer is inserted between them with an intermediate transmission axis direction. What is the new intensity of the light emerging from the final polarizer in Part E if a third polarizer (Polarizer A in the figure (Part F figure) ), whose transmission axis is offset 45 from each of the others, is inserted between the original two polarizers?

I thought that becuase the first polarizer and the third polarizer are perpendicular to each other, they would form an intensity of zero regardless of whatever angle the second polarizer is positioned at in the misddle. I dont understand this.

Any help?

25.46 Mastering Physics part B,C vs 25.44

to find the intensity, I used:
I1= c (8.85*10^-12) E^2 ---> part B
I2= c * B^2 /(4pi*10^-7) ---> part C

and finally got it right

but in the problem 25.44,
the Imax is given and Emax is asked
when I used the formula above I got it wrong, turned out needed to multiply by sqrt2 to get max value.

which formula is correct?

thanks

RLC lab (part 2: the capacitor)

when we're measuring the voltage of generator and the voltage across the resistor, why dont the waves line up?


Thanks

Friday, April 23, 2010

RL and RC circuit

why:

when an inductor and a resistor are connected in series, the voltage across the resistor after certain amount of time is V0 (1-e^t/time constant)

when a capacitor and a resistor are connected in series, the voltage across the resistor after certain amount of time is V0 (e^t/time constant)

He went over these in class but I forget

Please help
Thanks

Sorry it's out of topic

Sorry out of topic. But did anyone take my RLC lab by accident?
I really need that

thanks

Monday, April 19, 2010

To Study for the Exam

Doing problems that are harder than what is on the exam can STILL help you! Even if you get totally lost and confused half way through the problem this is IN FACT better than doing 20 easy problems you already knew how to do to begin with! Make sure you know:
1) How particles behave in the prescience(sp?) of a magnetic field (how does that differ if the charge is positive or negative)?
2) What kind of magnetic fields are produced from different sources (i.e. a current carrying wire, a moving charge, ect.)
3. How to do simple polarization problems
4. RC, RL, RLC circuit problems
5. AC circuit problems, WITH PHASORS!!!
6. Motional EMF (the thingy with the moving rectangle of wire into a magnetic field)
7. Calculating the magnitude and DIRECTION of the magnetic field from any source
8. the relationship (both VECTOR and otherwise) between the E and B-fields in a EM-wave

Mmmmmm.....how did I come up with this list?.......(that is a rhetorical question, please do not answer)

Friday, April 16, 2010

Question 25.44 Mastering Physics

The question asks:

What is the maximum value of the electric field in an electromagnetic wave whose maximum intensity is 5.50 {\rm W/m^{2}}?
I know that I=C*e_o*E^2

(e_o is my way of saying epsilon not)

So I rearranged to solve for E

E=sqrt(I/c*e_o)

The answer I get is 45.5.... but its not correct?

Any Suggestions?

Thursday, April 15, 2010

vector diagram

cant figure out how to use that phaser diagram. How do i draw on it, and those two arrows that you are given to start off with dont move?

Conceptual Question

When Maxwell proved that c, the speed of light, was equal to 1/sqrt(epsilon_0 x mu_0) he had proven something monumental. Why?

Friday, April 9, 2010

Helpful?

Does anyone ever go on this blog anymore? Are you guys finding it helpful or is it not really working out? This is the first semester I am trying this out on and so I am wondering if anyone is really using it or not. Let me know. Thanks

Saturday, March 27, 2010

Friday, March 26, 2010

22.10

A 6.60 uC particle moves through a region of space where an electric field of magnitude 1200 N/C points in the positive x direction and a magnetic field of magnitude 1.02T points in the positive z direction.

If the net force acting on the particle is 6.21x10^-3 N, in the positive x direction, find the components of the particle's velocity. Assume the particle's velocity is the x-y plane.

vx , vy , vz =

I just know we use the formulas:

F =qE
F=qvB

I can't figure how to get the velocities for the 3 components...

Sunday, March 21, 2010

MP 21.62 part B (Assignment 4 #16) Practice Problem

The capacitor in an RC circuit (R = 140 \Omega, C = 40 \mu F) is initially uncharged.
PartB
Find the current in the circuit one time constant (\tau = RC) after the circuit is connected to a 9.0-{\rm V} battery.

I use this but didnt work

I = V/R (1-e^-t/RC)

When the problem said one time constant, does that always mean that t/RC = 1, or only the RC is equal to 1?

I think Prof gave the similar problem on the last quiz, where it asked the current after 2 time constant. And he just plugged the "2" into t/RC

Questions 1 to 6 from Friday's SI Session - Test 1 Prep

What were the answers for the 6 questions rom Friday's SI session for Test 1 Prep. I tried to solve the problems; however, I don't know if I did them correctly. So does anyone have the answers so I can compare? Thank you.

MP 21.48 part E (Assignment 3 #12) Practice Problem

Calculate the potential at point D.

I dont really understand what the figure is all about. And, it says that point A is grounded. What does that mean? Does it affect any other point? Can I know the potential at point B?

Thanks.

Saturday, March 20, 2010

21.36 & 21.92

I'm having trouble figuring out 21.36 and 21.92 of the homework.

21.36
When two resistors, R_1 and R_2, are connected in series across a 6.0V battery, the potential difference across R_1 is 4.1 V. When R_1 and R_2 are connected in parallel to the same battery, the current through R_2 is 0.46 A.
Fine values of R_1 and R_2.



21.92
The equivalent resistance between points A and B of the resistors shown in the figure is 25 ohms. Find the value of Resistance R.

I'm having a hard time because there's no voltage given. Am I supposed to incorporate Kirchoff's rules somehow?

Checking the Blog

Just a suggestion for those of you who are utilizing the blog, you can do the following to make sure you are notified when I post a comment(answer) to your questions:
1. After publishing a new post click on the "Comment" button as if you are going to comment on your post. Then underneath the text box there should be a link called "Subscribe by email" if you click this then anytime anyone comments on that particular post the blog will send you an email with the comment in it.
2. you can become a follower of the blog which I believe makes the blog email you anytime the blog is updated by anyone.

For more info you can just comment to this post.

Dante

Friday, March 19, 2010

Equipotetial problem on Wed, 3-17

Dante,
I am still questioning about the problem that Prof gave on wednesday, the topomap.
The question was how much work would it take to move +1.o micro coulomb of charge from A to B (where the potential of B is higher than that of A by 4V).
I remember you explaining it and the sign is negative?
Also read p.668 on the textbook, it says "positive charges accelerate in the direction of decreasing electric potential".
Does that mean that if it goes in the direction of increasing V, it decelerates? then the work is negative?
When work is needed, is the sign negative? or positive?

Signs always confuse me. I have problem with the sign of electric potential and potential energy as well.
Please help.

thanks

DC CIRCUITS LAB--setting up the circuit

What do we need to know in order not to blow the fuse? Anything else other than these?
And please correct me if I'm wrong.

1.) Red goes to red (positive) and black goes to black (negative)
2.) Ampmeter in series(part of circuit), voltmeter in paralel (just touch whats being measured)
3.) Red goes to white when measuring the current. When do we use the yellow?
4.) Turn the knob to 200mA. What number do we use when measuring V and R?


Thanks

Monday, March 15, 2010

4b hw 4

A clyndrical shell of uniform charge density P0 lies in a region a .<.r<.b. find the E-field in all regions and graph E vs. r.- its a cylinder with a smaller cylinder as its hole. (couldnt get the picture up)

Thursday, March 11, 2010

FOR NEWCOMERS!

If you have not been to this blog before please read both the "Instructions" and "Welcome!" posts which are under the "February" folder in the "Blog Archive" on the right side. These two posts will tell you how I want you to post questions and answers!!
Thanks,
Dante

Remember the color patterns on the resistors in lab?

Assignment Due Date

Hey everyone! I think the prof accidentally set the due date for Mastering Physics homework for Saturday at 9am instead of pm.. just a heads up!!

21.66 - RC circuit

for this problem do we just plug in the values into the formula:

q(t) = C(emf) (1 - e^ (-t/T) ??

for some reason it isn't working out for me..

emf = 9V
R = 50.0 k Olms
C - 140 mF

it wants to find the time (t) it takes to charge the capacitor to 5.0V.

Tuesday, March 9, 2010

20.52

what is dielectric strength? how is it suppose to fit into problem 20.52? (first problem in hw 3)

Saturday, March 6, 2010

4b problem from Tatoian's hw-3 #5 and #9

5.A Water drop of radius 2.0 micro-meters is suspended in the atmosphere where the E-field's value is 500 V/m . How many electrons are there?


9.A 120-keV electron is fired directly toward a flat large plastic sheet with a charge density of -2 micro C/m^2. From what distance must the electron be fired if it is just to fail to strike the sheet?

Friday, March 5, 2010

20.5

If accelerating plates are 1.05cm apart and have a potential difference of 2.05 x 10^4 V, find the change in electric potential energy for an electron that moves from one accelerating plate to the other.

don't know why i cant do this one, i've exhausted all my attempts except for one..

For all of the Bio Majors

Energy of a Capacitor

A parallel plate vacuum capacitor is connected to a battery and charged until the stored electric energy is U. The batter is removed and then a dielectric material with dielectric constant k is insterted into the capacitor, filling the space between the places. Finally, the capacitor is fully discharged through a resistor (which is connected across the capacitor terminals)

a. )Find "Ur " the energy dissipated in the resistor Express your answer in terms of "U" and other given quantities.

b.) Consider the same situation as in the previous part, except that the charging battery remains connected while the dielectric is inserted. The battery is then disconnected and the capacitor is discharged. For this situation, what is "Ur" , the energy dissipated in the resistor. Express the answer in therms of U and other given quantities.

im a little confused, the resistor is throwing me off. How do i approach this problem?? How do i answer these in terms of "U"?

Thursday, March 4, 2010

Homework Conceptual Question 20.4

The question says:

A proton is released from rest in a region of space with a nonzero electric field. As the proton moves, does the electric potential energy of the system increase, decrease, or stay the same? Explain

Through trial and error I know that it decreases ;)

I just dont know why??

help?

Sunday, February 28, 2010

Practice Problem 19.48 Mastering Physics

The question says:

The electric field between the plates of a parallel-plate capacitor is horizontal, uniform, and has a magnitude E. A small object with a charge of -2.12 \mu C is attached to the string. The tension in the string is 0.310 N, and the angle it makes with the vertical is 12 ^\circ.

What is the mass of the object?

What is the magnitude of the electric field?


Not sure where to even begin here.... help?

Saturday, February 27, 2010

Testing Testing

1..2..3... ;)

Instructions

Instructions on how to ask and leave feedback on the blog:
1. Each post that is made by anyone should be a separate question. For example, if someone posts "I don't know how to do #3 on the homework" don't reply to them by adding another post REPLY BY LEAVING A COMMENT! This way each question will have its own post.

2. When you post something choose the "Compose" tab NOT the "Edit HTML" tab!

3. Each time you post a new question there should be a text box on the bottom left of the posting window called "Label". Write in either the question # or a small description of the question you are asking so other people can find it if they search. For example: Assignment #3 "Two point charges" or Ch.19 Problem #3

4. If you are writing a post about a masteringphysics homework question be sure to write the NAME of the question along with both the assignment number and which part (A,B,ect.) in the "Labels for this post" text box

Thanks guys,
Dante

Welcome!

Welcome to the Physics 2B Blog!
Using this blog you will be able to be in direct communication with both your fellow students and your Physics 2B SI Dante. This is the first time I am using a blog for this purpose so please give me feedback as to the blog's effectiveness in helping you with your schoolwork. Please feel free to post any questions. All of your classmates' emails should be viewable also so that you can email each other and me. If there are any questions about the blog please feel free to ask me.

Dante